Showing that $[0, omega_1]$ is compact. [duplicate]











up vote
0
down vote

favorite













This question already has an answer here:




  • First uncountable ordinal

    1 answer




I want to show that $[0, omega_1]$ is compact, where $omega_1$ is the least uncountable ordinal, and I have just been introduced to the concepts of ordinals.



The tips I have seen to showing this is that $[0, omega_1]$:




  • contains a maximal element

  • there are no infinite strictly decreasing sequences of ordinals.


Take an open cover of the desired set. My thoughts are that some open set must contain $omega_1$, which in turn contains all ordinals smaller than it by the definition of an ordinal, and so this open set is a cover of the entire space. Any further insights appreciated.










share|cite|improve this question















marked as duplicate by Asaf Karagila general-topology
Users with the  general-topology badge can single-handedly close general-topology questions as duplicates and reopen them as needed.

StackExchange.ready(function() {
if (StackExchange.options.isMobile) return;

$('.dupe-hammer-message-hover:not(.hover-bound)').each(function() {
var $hover = $(this).addClass('hover-bound'),
$msg = $hover.siblings('.dupe-hammer-message');

$hover.hover(
function() {
$hover.showInfoMessage('', {
messageElement: $msg.clone().show(),
transient: false,
position: { my: 'bottom left', at: 'top center', offsetTop: -7 },
dismissable: false,
relativeToBody: true
});
},
function() {
StackExchange.helpers.removeMessages();
}
);
});
});
Nov 23 at 8:50


This question has been asked before and already has an answer. If those answers do not fully address your question, please ask a new question.















  • An alternative way is to use that for a countable ordinal $beta$, the set $[0,beta]$ is compact in the order topology using some criterion for compactness in ordered spaces, ifyou know such theorems.
    – Henno Brandsma
    Nov 23 at 4:56










  • @HennoBrandsma I have a theorem in my notes (with a proof) that states : Let $X$ be a simply ordered set with the least upper bound property with respect to the order topology. Then each closed interval is compact. Does this apply? I guess really I am asking if the least upper bound property is present here?
    – IntegrateThis
    Nov 23 at 5:00








  • 1




    Yes, you could use that theorem to show that for all ordinals $[0,beta]$ is compact. A subset of ordinals wtih an upper bound has a sup, as this we can take the minimum of the set of upperbounds, which exists as all non-empty sets of ordinals have a minimum (equivalent to your property it hss no infinite strictly decreasing subsets).
    – Henno Brandsma
    Nov 23 at 5:11

















up vote
0
down vote

favorite













This question already has an answer here:




  • First uncountable ordinal

    1 answer




I want to show that $[0, omega_1]$ is compact, where $omega_1$ is the least uncountable ordinal, and I have just been introduced to the concepts of ordinals.



The tips I have seen to showing this is that $[0, omega_1]$:




  • contains a maximal element

  • there are no infinite strictly decreasing sequences of ordinals.


Take an open cover of the desired set. My thoughts are that some open set must contain $omega_1$, which in turn contains all ordinals smaller than it by the definition of an ordinal, and so this open set is a cover of the entire space. Any further insights appreciated.










share|cite|improve this question















marked as duplicate by Asaf Karagila general-topology
Users with the  general-topology badge can single-handedly close general-topology questions as duplicates and reopen them as needed.

StackExchange.ready(function() {
if (StackExchange.options.isMobile) return;

$('.dupe-hammer-message-hover:not(.hover-bound)').each(function() {
var $hover = $(this).addClass('hover-bound'),
$msg = $hover.siblings('.dupe-hammer-message');

$hover.hover(
function() {
$hover.showInfoMessage('', {
messageElement: $msg.clone().show(),
transient: false,
position: { my: 'bottom left', at: 'top center', offsetTop: -7 },
dismissable: false,
relativeToBody: true
});
},
function() {
StackExchange.helpers.removeMessages();
}
);
});
});
Nov 23 at 8:50


This question has been asked before and already has an answer. If those answers do not fully address your question, please ask a new question.















  • An alternative way is to use that for a countable ordinal $beta$, the set $[0,beta]$ is compact in the order topology using some criterion for compactness in ordered spaces, ifyou know such theorems.
    – Henno Brandsma
    Nov 23 at 4:56










  • @HennoBrandsma I have a theorem in my notes (with a proof) that states : Let $X$ be a simply ordered set with the least upper bound property with respect to the order topology. Then each closed interval is compact. Does this apply? I guess really I am asking if the least upper bound property is present here?
    – IntegrateThis
    Nov 23 at 5:00








  • 1




    Yes, you could use that theorem to show that for all ordinals $[0,beta]$ is compact. A subset of ordinals wtih an upper bound has a sup, as this we can take the minimum of the set of upperbounds, which exists as all non-empty sets of ordinals have a minimum (equivalent to your property it hss no infinite strictly decreasing subsets).
    – Henno Brandsma
    Nov 23 at 5:11















up vote
0
down vote

favorite









up vote
0
down vote

favorite












This question already has an answer here:




  • First uncountable ordinal

    1 answer




I want to show that $[0, omega_1]$ is compact, where $omega_1$ is the least uncountable ordinal, and I have just been introduced to the concepts of ordinals.



The tips I have seen to showing this is that $[0, omega_1]$:




  • contains a maximal element

  • there are no infinite strictly decreasing sequences of ordinals.


Take an open cover of the desired set. My thoughts are that some open set must contain $omega_1$, which in turn contains all ordinals smaller than it by the definition of an ordinal, and so this open set is a cover of the entire space. Any further insights appreciated.










share|cite|improve this question
















This question already has an answer here:




  • First uncountable ordinal

    1 answer




I want to show that $[0, omega_1]$ is compact, where $omega_1$ is the least uncountable ordinal, and I have just been introduced to the concepts of ordinals.



The tips I have seen to showing this is that $[0, omega_1]$:




  • contains a maximal element

  • there are no infinite strictly decreasing sequences of ordinals.


Take an open cover of the desired set. My thoughts are that some open set must contain $omega_1$, which in turn contains all ordinals smaller than it by the definition of an ordinal, and so this open set is a cover of the entire space. Any further insights appreciated.





This question already has an answer here:




  • First uncountable ordinal

    1 answer








general-topology compactness ordinals well-orders






share|cite|improve this question















share|cite|improve this question













share|cite|improve this question




share|cite|improve this question








edited Nov 23 at 8:48









Asaf Karagila

301k32422753




301k32422753










asked Nov 23 at 3:40









IntegrateThis

1,7081717




1,7081717




marked as duplicate by Asaf Karagila general-topology
Users with the  general-topology badge can single-handedly close general-topology questions as duplicates and reopen them as needed.

StackExchange.ready(function() {
if (StackExchange.options.isMobile) return;

$('.dupe-hammer-message-hover:not(.hover-bound)').each(function() {
var $hover = $(this).addClass('hover-bound'),
$msg = $hover.siblings('.dupe-hammer-message');

$hover.hover(
function() {
$hover.showInfoMessage('', {
messageElement: $msg.clone().show(),
transient: false,
position: { my: 'bottom left', at: 'top center', offsetTop: -7 },
dismissable: false,
relativeToBody: true
});
},
function() {
StackExchange.helpers.removeMessages();
}
);
});
});
Nov 23 at 8:50


This question has been asked before and already has an answer. If those answers do not fully address your question, please ask a new question.






marked as duplicate by Asaf Karagila general-topology
Users with the  general-topology badge can single-handedly close general-topology questions as duplicates and reopen them as needed.

StackExchange.ready(function() {
if (StackExchange.options.isMobile) return;

$('.dupe-hammer-message-hover:not(.hover-bound)').each(function() {
var $hover = $(this).addClass('hover-bound'),
$msg = $hover.siblings('.dupe-hammer-message');

$hover.hover(
function() {
$hover.showInfoMessage('', {
messageElement: $msg.clone().show(),
transient: false,
position: { my: 'bottom left', at: 'top center', offsetTop: -7 },
dismissable: false,
relativeToBody: true
});
},
function() {
StackExchange.helpers.removeMessages();
}
);
});
});
Nov 23 at 8:50


This question has been asked before and already has an answer. If those answers do not fully address your question, please ask a new question.














  • An alternative way is to use that for a countable ordinal $beta$, the set $[0,beta]$ is compact in the order topology using some criterion for compactness in ordered spaces, ifyou know such theorems.
    – Henno Brandsma
    Nov 23 at 4:56










  • @HennoBrandsma I have a theorem in my notes (with a proof) that states : Let $X$ be a simply ordered set with the least upper bound property with respect to the order topology. Then each closed interval is compact. Does this apply? I guess really I am asking if the least upper bound property is present here?
    – IntegrateThis
    Nov 23 at 5:00








  • 1




    Yes, you could use that theorem to show that for all ordinals $[0,beta]$ is compact. A subset of ordinals wtih an upper bound has a sup, as this we can take the minimum of the set of upperbounds, which exists as all non-empty sets of ordinals have a minimum (equivalent to your property it hss no infinite strictly decreasing subsets).
    – Henno Brandsma
    Nov 23 at 5:11




















  • An alternative way is to use that for a countable ordinal $beta$, the set $[0,beta]$ is compact in the order topology using some criterion for compactness in ordered spaces, ifyou know such theorems.
    – Henno Brandsma
    Nov 23 at 4:56










  • @HennoBrandsma I have a theorem in my notes (with a proof) that states : Let $X$ be a simply ordered set with the least upper bound property with respect to the order topology. Then each closed interval is compact. Does this apply? I guess really I am asking if the least upper bound property is present here?
    – IntegrateThis
    Nov 23 at 5:00








  • 1




    Yes, you could use that theorem to show that for all ordinals $[0,beta]$ is compact. A subset of ordinals wtih an upper bound has a sup, as this we can take the minimum of the set of upperbounds, which exists as all non-empty sets of ordinals have a minimum (equivalent to your property it hss no infinite strictly decreasing subsets).
    – Henno Brandsma
    Nov 23 at 5:11


















An alternative way is to use that for a countable ordinal $beta$, the set $[0,beta]$ is compact in the order topology using some criterion for compactness in ordered spaces, ifyou know such theorems.
– Henno Brandsma
Nov 23 at 4:56




An alternative way is to use that for a countable ordinal $beta$, the set $[0,beta]$ is compact in the order topology using some criterion for compactness in ordered spaces, ifyou know such theorems.
– Henno Brandsma
Nov 23 at 4:56












@HennoBrandsma I have a theorem in my notes (with a proof) that states : Let $X$ be a simply ordered set with the least upper bound property with respect to the order topology. Then each closed interval is compact. Does this apply? I guess really I am asking if the least upper bound property is present here?
– IntegrateThis
Nov 23 at 5:00






@HennoBrandsma I have a theorem in my notes (with a proof) that states : Let $X$ be a simply ordered set with the least upper bound property with respect to the order topology. Then each closed interval is compact. Does this apply? I guess really I am asking if the least upper bound property is present here?
– IntegrateThis
Nov 23 at 5:00






1




1




Yes, you could use that theorem to show that for all ordinals $[0,beta]$ is compact. A subset of ordinals wtih an upper bound has a sup, as this we can take the minimum of the set of upperbounds, which exists as all non-empty sets of ordinals have a minimum (equivalent to your property it hss no infinite strictly decreasing subsets).
– Henno Brandsma
Nov 23 at 5:11






Yes, you could use that theorem to show that for all ordinals $[0,beta]$ is compact. A subset of ordinals wtih an upper bound has a sup, as this we can take the minimum of the set of upperbounds, which exists as all non-empty sets of ordinals have a minimum (equivalent to your property it hss no infinite strictly decreasing subsets).
– Henno Brandsma
Nov 23 at 5:11












2 Answers
2






active

oldest

votes

















up vote
3
down vote



accepted










We can show for an arbitrary ordinal $alpha$ that $[0,alpha]$ is compact with the order topology.



Let $mathcal U$ be an open cover of $[0,alpha]$, and let $beta$ be the smallest ordinal such that $[beta,alpha]$ is covered by a finite subcover of $mathcal U$. (Certainly there are some $beta$s with this property, such as $alpha$ itself, so there must be a smallest one).



Now if $beta$ is a successor ordinal $beta=delta+1$, then there must be an element of $mathcal U$ that contains $delta$, and adding that to our finite cover of $[beta,alpha]$ produces a finite cover of $[delta,alpha]$, a contradiction.



On the other hand, if $beta$ is a limit ordinal, then every open set that contains $beta$ must also contain some $delta<beta$, and therefore our finite subcover of $[beta,alpha]$ is already a cover of $[delta,alpha]$. Again a contradiction.



The only possibility remaining is that $beta=0$.






share|cite|improve this answer




























    up vote
    1
    down vote













    Suppose that $mathcal{U}$ is an open cover of $[0,omega_1]$ by basic open sets of the order topology (that's the topology we are putting on this ordered set), and suppose for a contradiction that it has no finite subcover. (It suffices to consider bcovers by base elements, as is easy to see in all topological spaces, if you know Alexander's subbase theorem, the proof will be easier even).



    Some element of the cover has to contain the maximale element $omega_1$, and so some $U_0 in mathcal{U}$ must be of the form $(beta_0, omega_1]$ (such are the basic open subsets in the order topology that contain the maximal element).



    Also, $0$ must be covered so there is some $U'_0 in mathcal{U}$ of the form $[0,alpha_0)$ that contains $0$.



    Now we know that $U_0, U'_0$ cannot be a finite subcover (which would happen when $alpha_0 > beta_0$), and so that some $U_1 in mathcal{U}$ must cover $beta_0$, say $beta_0 in U_1 = (alpha_2, beta_1)$, which implies that $alpha_2 < beta_0 < beta_1$.



    Now consider what happens if the sets $U_0, U'_0, U_1$ do not form a subcover yet, find a new element to cover etc., and define an infinite decreasing sequence of ordinals that way.






    share|cite|improve this answer






























      2 Answers
      2






      active

      oldest

      votes








      2 Answers
      2






      active

      oldest

      votes









      active

      oldest

      votes






      active

      oldest

      votes








      up vote
      3
      down vote



      accepted










      We can show for an arbitrary ordinal $alpha$ that $[0,alpha]$ is compact with the order topology.



      Let $mathcal U$ be an open cover of $[0,alpha]$, and let $beta$ be the smallest ordinal such that $[beta,alpha]$ is covered by a finite subcover of $mathcal U$. (Certainly there are some $beta$s with this property, such as $alpha$ itself, so there must be a smallest one).



      Now if $beta$ is a successor ordinal $beta=delta+1$, then there must be an element of $mathcal U$ that contains $delta$, and adding that to our finite cover of $[beta,alpha]$ produces a finite cover of $[delta,alpha]$, a contradiction.



      On the other hand, if $beta$ is a limit ordinal, then every open set that contains $beta$ must also contain some $delta<beta$, and therefore our finite subcover of $[beta,alpha]$ is already a cover of $[delta,alpha]$. Again a contradiction.



      The only possibility remaining is that $beta=0$.






      share|cite|improve this answer

























        up vote
        3
        down vote



        accepted










        We can show for an arbitrary ordinal $alpha$ that $[0,alpha]$ is compact with the order topology.



        Let $mathcal U$ be an open cover of $[0,alpha]$, and let $beta$ be the smallest ordinal such that $[beta,alpha]$ is covered by a finite subcover of $mathcal U$. (Certainly there are some $beta$s with this property, such as $alpha$ itself, so there must be a smallest one).



        Now if $beta$ is a successor ordinal $beta=delta+1$, then there must be an element of $mathcal U$ that contains $delta$, and adding that to our finite cover of $[beta,alpha]$ produces a finite cover of $[delta,alpha]$, a contradiction.



        On the other hand, if $beta$ is a limit ordinal, then every open set that contains $beta$ must also contain some $delta<beta$, and therefore our finite subcover of $[beta,alpha]$ is already a cover of $[delta,alpha]$. Again a contradiction.



        The only possibility remaining is that $beta=0$.






        share|cite|improve this answer























          up vote
          3
          down vote



          accepted







          up vote
          3
          down vote



          accepted






          We can show for an arbitrary ordinal $alpha$ that $[0,alpha]$ is compact with the order topology.



          Let $mathcal U$ be an open cover of $[0,alpha]$, and let $beta$ be the smallest ordinal such that $[beta,alpha]$ is covered by a finite subcover of $mathcal U$. (Certainly there are some $beta$s with this property, such as $alpha$ itself, so there must be a smallest one).



          Now if $beta$ is a successor ordinal $beta=delta+1$, then there must be an element of $mathcal U$ that contains $delta$, and adding that to our finite cover of $[beta,alpha]$ produces a finite cover of $[delta,alpha]$, a contradiction.



          On the other hand, if $beta$ is a limit ordinal, then every open set that contains $beta$ must also contain some $delta<beta$, and therefore our finite subcover of $[beta,alpha]$ is already a cover of $[delta,alpha]$. Again a contradiction.



          The only possibility remaining is that $beta=0$.






          share|cite|improve this answer












          We can show for an arbitrary ordinal $alpha$ that $[0,alpha]$ is compact with the order topology.



          Let $mathcal U$ be an open cover of $[0,alpha]$, and let $beta$ be the smallest ordinal such that $[beta,alpha]$ is covered by a finite subcover of $mathcal U$. (Certainly there are some $beta$s with this property, such as $alpha$ itself, so there must be a smallest one).



          Now if $beta$ is a successor ordinal $beta=delta+1$, then there must be an element of $mathcal U$ that contains $delta$, and adding that to our finite cover of $[beta,alpha]$ produces a finite cover of $[delta,alpha]$, a contradiction.



          On the other hand, if $beta$ is a limit ordinal, then every open set that contains $beta$ must also contain some $delta<beta$, and therefore our finite subcover of $[beta,alpha]$ is already a cover of $[delta,alpha]$. Again a contradiction.



          The only possibility remaining is that $beta=0$.







          share|cite|improve this answer












          share|cite|improve this answer



          share|cite|improve this answer










          answered Nov 23 at 5:08









          Henning Makholm

          236k16300534




          236k16300534






















              up vote
              1
              down vote













              Suppose that $mathcal{U}$ is an open cover of $[0,omega_1]$ by basic open sets of the order topology (that's the topology we are putting on this ordered set), and suppose for a contradiction that it has no finite subcover. (It suffices to consider bcovers by base elements, as is easy to see in all topological spaces, if you know Alexander's subbase theorem, the proof will be easier even).



              Some element of the cover has to contain the maximale element $omega_1$, and so some $U_0 in mathcal{U}$ must be of the form $(beta_0, omega_1]$ (such are the basic open subsets in the order topology that contain the maximal element).



              Also, $0$ must be covered so there is some $U'_0 in mathcal{U}$ of the form $[0,alpha_0)$ that contains $0$.



              Now we know that $U_0, U'_0$ cannot be a finite subcover (which would happen when $alpha_0 > beta_0$), and so that some $U_1 in mathcal{U}$ must cover $beta_0$, say $beta_0 in U_1 = (alpha_2, beta_1)$, which implies that $alpha_2 < beta_0 < beta_1$.



              Now consider what happens if the sets $U_0, U'_0, U_1$ do not form a subcover yet, find a new element to cover etc., and define an infinite decreasing sequence of ordinals that way.






              share|cite|improve this answer



























                up vote
                1
                down vote













                Suppose that $mathcal{U}$ is an open cover of $[0,omega_1]$ by basic open sets of the order topology (that's the topology we are putting on this ordered set), and suppose for a contradiction that it has no finite subcover. (It suffices to consider bcovers by base elements, as is easy to see in all topological spaces, if you know Alexander's subbase theorem, the proof will be easier even).



                Some element of the cover has to contain the maximale element $omega_1$, and so some $U_0 in mathcal{U}$ must be of the form $(beta_0, omega_1]$ (such are the basic open subsets in the order topology that contain the maximal element).



                Also, $0$ must be covered so there is some $U'_0 in mathcal{U}$ of the form $[0,alpha_0)$ that contains $0$.



                Now we know that $U_0, U'_0$ cannot be a finite subcover (which would happen when $alpha_0 > beta_0$), and so that some $U_1 in mathcal{U}$ must cover $beta_0$, say $beta_0 in U_1 = (alpha_2, beta_1)$, which implies that $alpha_2 < beta_0 < beta_1$.



                Now consider what happens if the sets $U_0, U'_0, U_1$ do not form a subcover yet, find a new element to cover etc., and define an infinite decreasing sequence of ordinals that way.






                share|cite|improve this answer

























                  up vote
                  1
                  down vote










                  up vote
                  1
                  down vote









                  Suppose that $mathcal{U}$ is an open cover of $[0,omega_1]$ by basic open sets of the order topology (that's the topology we are putting on this ordered set), and suppose for a contradiction that it has no finite subcover. (It suffices to consider bcovers by base elements, as is easy to see in all topological spaces, if you know Alexander's subbase theorem, the proof will be easier even).



                  Some element of the cover has to contain the maximale element $omega_1$, and so some $U_0 in mathcal{U}$ must be of the form $(beta_0, omega_1]$ (such are the basic open subsets in the order topology that contain the maximal element).



                  Also, $0$ must be covered so there is some $U'_0 in mathcal{U}$ of the form $[0,alpha_0)$ that contains $0$.



                  Now we know that $U_0, U'_0$ cannot be a finite subcover (which would happen when $alpha_0 > beta_0$), and so that some $U_1 in mathcal{U}$ must cover $beta_0$, say $beta_0 in U_1 = (alpha_2, beta_1)$, which implies that $alpha_2 < beta_0 < beta_1$.



                  Now consider what happens if the sets $U_0, U'_0, U_1$ do not form a subcover yet, find a new element to cover etc., and define an infinite decreasing sequence of ordinals that way.






                  share|cite|improve this answer














                  Suppose that $mathcal{U}$ is an open cover of $[0,omega_1]$ by basic open sets of the order topology (that's the topology we are putting on this ordered set), and suppose for a contradiction that it has no finite subcover. (It suffices to consider bcovers by base elements, as is easy to see in all topological spaces, if you know Alexander's subbase theorem, the proof will be easier even).



                  Some element of the cover has to contain the maximale element $omega_1$, and so some $U_0 in mathcal{U}$ must be of the form $(beta_0, omega_1]$ (such are the basic open subsets in the order topology that contain the maximal element).



                  Also, $0$ must be covered so there is some $U'_0 in mathcal{U}$ of the form $[0,alpha_0)$ that contains $0$.



                  Now we know that $U_0, U'_0$ cannot be a finite subcover (which would happen when $alpha_0 > beta_0$), and so that some $U_1 in mathcal{U}$ must cover $beta_0$, say $beta_0 in U_1 = (alpha_2, beta_1)$, which implies that $alpha_2 < beta_0 < beta_1$.



                  Now consider what happens if the sets $U_0, U'_0, U_1$ do not form a subcover yet, find a new element to cover etc., and define an infinite decreasing sequence of ordinals that way.







                  share|cite|improve this answer














                  share|cite|improve this answer



                  share|cite|improve this answer








                  edited Nov 23 at 5:08

























                  answered Nov 23 at 4:54









                  Henno Brandsma

                  103k345113




                  103k345113















                      Popular posts from this blog

                      How do I know what Microsoft account the skydrive app is syncing to?

                      When does type information flow backwards in C++?

                      Grease: Live!